LSAT and Law School Admissions Forum

Get expert LSAT preparation and law school admissions advice from PowerScore Test Preparation.

 Administrator
PowerScore Staff
  • PowerScore Staff
  • Posts: 8917
  • Joined: Feb 02, 2011
|
#32021
Complete Question Explanation
(The complete setup for this game can be found here: lsat/viewtopic.php?t=15203)

The correct answer choice is (B)

This Global question asks us to determine what cannot be the complete assignment of variables to L. L is the group we know the least about, which compounds the difficulty of this question. Once again, the best course of action may well be to skip the question and tackle some of the local questions in the game first. Each local question will provide an opportunity to identify what could be a complete assignment of variables to L, which may, in turn, help eliminate at least some of the incorrect answer choices.

Note, also, that each answer choice contains a list of two variables. For such a list to represent a complete and accurate list of the variables assigned to L, our solution will need to conform to a 2-2-2 distribution, since M must also have exactly two variables assigned to it (first rule).

Answer choice (A): This answer choice is incorrect, because R and V could be the complete and accurate list of variables assigned to L (consider the previously-shown Possibility 1B, for instance).

Answer choice (B): This is the correct answer choice. If R and Z were the variables assigned to L, that would mean that Y cannot be assigned to P, because Z is not assigned to P. Thus, Y would be assigned to M. However, we also need to separate Y and V in compliance with the third rule, making it impossible to accommodate the ST block as required by the second rule:
PT79_Game_#3_#14_diagram 1.png
Answer choice (C): This answer choice is incorrect, because S and T could be the complete and accurate list of variables assigned to L. See Possibility 1A.

Answer choice (D): This answer choice is incorrect, because V and Z could be the complete and accurate list of variables assigned to L. See Question #16 (further illustrating the potential value of saving this question for later).

Answer choice (E): This answer choice is incorrect, because Y and Z could be the complete and accurate list of variables assigned to L. For instance, the following solution does not violate any of the rules in the game:
PT79_Game_#3_#14_diagram 2.png
You do not have the required permissions to view the files attached to this post.

Get the most out of your LSAT Prep Plus subscription.

Analyze and track your performance with our Testing and Analytics Package.